You are on page 1of 17

e­Prac­CAT # 03

ANALYSIS

About Prac­CATs 
Hi!  Welcome  to  PT’s  trendsetting  e­Prac­CAT  series  that  provides  the  cutting  edge  to  all  test 
takers with its innovative approach towards paper setting styles, question patterns, and inductive 
& deductive reasoning skill requirements. 

The  e­Prac­CAT  series  has  all  the  features  that  make  it  unique  in  content  and  approach,  thus 
making it the best tool in the country for students aiming at the best B Schools. 

The e­Prac­CATs by PT Education will provide you various possible simulations of the actual CAT. 
The  multiple paper patterns, differential  marking,  and  different  negative  markings  will  give  you 
an actual feel of the unpredictable CAT. 

Special features of the Prac­CAT series: 
1.  Innovative questions requiring high quality reasoning 
2.  Benchmark in the standards of questions asked 
3.  Interpretive ability tested to the maximum 
4.  All possible varieties of the CAT paper simulations 
5.  Detailed question by question analysis on your performance available on  the PT 
Website for every student. 
6.  Authentic AIRs 
7.  Gyan Dhara for each student (also for students who register for only the e­Prac­ 
CAT series, restricted to e­Prac­CATs only) 
8.  Doubts clearing  services and personal  counseling  sessions  for  all students  (with 
prior appointments) 

The test that you just took was an attempt to help you gear up for the coming test(s). Different 
variations in terms of toughness of tests, total questions etc. should prepare you for your dream 
career. So, remind yourself again... 

Kar Ke Dikhayenge!

Corp. Office: Delhi  Regd. Office: Indore 
PT centres spread across India ~ Established 1993 

Our motto “Kar Ke Dikhayenge” is delivered through our 
unique Technology Driven Process Engine (TDpro engine). 
Email: pinnacle@PTeducation.com  Web: www.PTeducation.com

IC : PTpnrps03  1 of (17) 
1. Ans.(3). If we look up paragraph 3, we can understand that the author has a grudge against tacking the story up with the gameplay. Why does he think 
so?  Because as he  explains further, the storyline  moved previously with one motivator,  which  was "fun".  Even  in the newer version of the game, this 
inducement  could have done its part. There was no actual need for the added narrative.  Now, the answer to question would require us to understand 
what  is  "blatant".  It  means  something  that  is  completely  obvious  or  vehemently  loud.  The  reason  why  the  storyline  justifies  this  adjective  is  that 
according to the author it is the most visible feature because of its uselessness. Thus, option (3) is correct because it encompasses all the major points. 
Option  (1) misses  out  the  unsuccessfulness  of  the  storyline.  Option  (2)  is  an  innocuous  one  and  could  have  been chosen  given  there  was  a  lack  of 
better choice. Option (4) is not supported by the author. He may have mentioned that the narrative was redundant but he never says that it  shifts the 
focus away.  These are two different things. Option  (5) is not even close as it  is talking of improving the gameplay, when the question asks about the 
fallacies of the move. 
2. Ans. (1). Harrowing means "causing pain, or trouble". Option (2) is incorrect because intertwining means overlapping which isn't the intended meaning. 
Option (5) is wrong because levels is not what the author meant by courses. Among options (1), (3) and (4) option (1) is a better choice because when 
one is talking about the difficulty of the game what one means is that it is troublesome, and not traumatic or agonizing. These are higher degrees which 
should be avoided. 
3. Ans.(1). Read  the  first  line of the  passage and  it  becomes clear that  the  author  loves the  newer version  (Marble  Saga) over the  older one  (Marble 
Mania).  Also, throughout the passage, he maintains this tone. Options (3) and (4) are wrong as they ask for the removal of the narrative for the making 
of the choice. This, as is evident from the passage is not a necessary condition. Of course, the improvement of the controls is an enhancement in the 
later version, but this is mentioned as a sideline benefit. 
4. Ans.(1). The passage uses disarming as "Marble Saga  Kororinpa is disarming in its  simplicity". Here, the clear meaning is that Marble Saga has won 
over the critic's heart with its simplicity. Even the meaning of the word "disarming" is "to win over". 
5. Ans.(3). Option  (3) best  rounds up what the author has in  mind. Options  (1) and (2) are incorrect because they compare the  newer version with the 
older one; this is not required. Option (5) misses out the negative feedback. Option (4) is a close one but it forgets to mention the game's "already great 
formula". 
6. Ans.(5).  The  third  reason  given  in  the  passage  about  the  acceptance  of  relativism  talks  of accepting  different  reasons  or  logics  in  view  of  different 
cultures.  So, option (1)  is ruled out. Options (2) and (3) can be deduced from the  first paragraph. Option (4) implies the same as the other three  and 
says that there is no singular benchmark for absolute truth. Hence, the answer is option (5). 
7. Ans.(5).  A doctrine  is  self­refuting  if  its  truth  implies its falsehood .Option(1)  plays on the  definition  of relativism,  since  the  truth  value of a  particular 
statement is relative, it can at the same time be true or false with respect to different statements hence it clearly implies that relativism itself is false, at 
least relative to some standpoints. Option (2) is a clear example of self refutation given in the passage. Option (3) is stated in the opening paragraphs 
of the passage. 
8. Ans.(4). Options (1), (2) and (3) can be inferred from the second ,third and fourth reasons  for acceptance of cognitive relativism. Media here refers to 
the broader term that reflects all medium of expression, viz. Drama, Literature, etc. Option (5) is stated in the seventh and eighth paragraph. Option (4) 
is the only one that is not supported by the passage. 
9. Ans (4). Option(1) is not correct as this is the view of critics and not that of realists themselves. Option (2) is also wrong as it describes the perspective 
of certain 'sophisticated' relativists and not that of all relativists on the whole. Option (3) is wrong as the passage says "cognitive relativism is a thought 
to undermine………..",i.e. , not the view of realists themselves. Option (4) is right as it has been explicitly stated in the passage. 
10. Ans.(2).  Option  (1)  fails  because  it  brands all  unintelligent  people  to  be  the  degrader  of  relativism.  Option  (3)  speaks  about  who  do  not  represent 
relativism, while the paragraph was talking about who represent relativism incorrectly. Option (4) is close but covers only one part of the paragraph and 
leaves out what this activity leads to. Option (5) is incorrect as it intends to state information contrary to the paragraph. 
11. Ans.(4). If we look at the given statement in the passage, we  clearly see that author agrees to  it as being an issue, but an over­rated one and not as 
serious as is made out to be. Option (1) is given nowhere in the passage. Option (2) though has been given in the passage but is not the right answer 
because it does not  relate to the question. It is just a justification of how other issues have been overshadowed by global warming. Option  (3) shows 
how activists exaggerate things and does not convey the meaning of given, it is simply an example to prove it. Option (5) clearly does not fit in. 
12. Ans.(3). Immediately before the last statement, the author has said that popular media has given 300 problems arising out of global warming. Last line 
is a mockery of this and means that just the way people believed pope when he said witches caused deaths due to cold, similarly people today believe 
global warming as being the  "witch" that  is causing  all the problems. Although author believes that activists  sometimes misguide people he  does not 
relate  it to people's belief about  witches.  Nowhere has the author talked about superstition as being one of the major problems.  Hence, option (4)  is 
rejected. 
13. Ans.(2)  At first  glance  option(5)  seems to  be  right  as  all  others  have  been  mentioned  in  the  passage  but  on  reading  again  we  find  that though  the 
author has talked of counter­effects of global warming, nowhere does he say that the two will balance out each other's effects. Option (1) finds mention 
in the last line of paragraph 3. Option (3) has been justified in the last paragraph. Option (4) can be inferred from paragraph 4. 
14. Ans.(4) This is the correct answer as in the last two paragraphs, the author talks of investing funds wisely and equally  in all the major problems than 
just focusing  on global  warming.  This is on the  same lines as options (3) and (5) but  is more  appropriate as it  justifies them both.  Thus, (4)  is  an all­ 
inclusive answer. Options (1) and (2) are the assertions made by author as reasons causing the exaggeration of issue. He does not talk of eradicating 
them. 
15. Ans.(2). Arrant actually means "without qualification; used informally as (often pejorative) an intensifier". Clearly, option (2) is the correct one. We can 
solve this question by looking up the context in which this word has been used in the passage. 
16. Ans.(1). It is clear that all options are mentioned in the passage except option (1). This is contrary to what the passage says that 'Chickens and eggs 
raised on small, sustainable farms seem relatively expensive' in paragraph 2. 
17. Ans.(4). The line has to mean that after eating your first egg sandwich, the taste that lingers in the mouth is one that you remember and hence in every 
next purchase,  you try to have something  like it or  better than  it.  So, the eggs that  you  purchase, you try to have the ones that can provide the best 
possible taste. This is mentioned in option (4). All other options can be true if we visualise the line alone. But if taken in the context of the passage, only 
option (4) fits in. 
18. Ans.(3). The author is thrilled in the first paragraph to witness eggs being sold at $6. This reveals that this is a high price and yet there are people who 
are buying the eggs. This goes to show that there is a  change in the buying pattern of the consumers. This change becomes  evident as we  read the 
next few paragraphs. Thus, option (3)is the best answer. 
19. Ans.(3). Paragraph 1 states that the author viewed  $6 eggs as a  rarity.  He felt thrilled to see this. The answer to why he felt thrilled  can be given by 
answering why he considered the eggs as a rarity. He did that because he had never seen so expensive eggs being sold. This eliminates options (2), 
(4) and (5). Option (1) is cancelled out because the inability to sell eggs at $4 or $5 was not a discovery for the author as the option makes us believe.

2 of 17  PTpnrps03 
20. Ans.(4). Only option (4) is incorrect as the passage says "humane  standards for the animals" which means treating animals considerately and kindly, 
not humanly. 
21. Ans.(2).  The  passage  undoubtedly  talks  about  winemaking,  but  in  clear reference  to the  state  of  Burgundy.  Even  from  the first  paragraph  one  can 
deduce the author's motive that he wishes to examine the role played by Burgundy in the making of wine. Only option (2) speaks about this. 
22. Ans.(2). If all the grapes from differnet vineyards could provide similar (best) quality of the grapes they could be used to create a similar (best) quality of 
wine. Thus, option (2) seems the most plausible answer. Options (1) and (3) are redundant as they state something that the passage states. Option (4) 
has the flaw that the advantage enlisted has nothing to do with the development of winemaking. 
23. Ans.(3). This question  can be answered  only by understanding the text that follows  this line in the passage. On reading it we  can ascertain that the 
author is  speaking  about  the  experience  a  person  has after  drinking  Burgundy wine. The  person  savours  the  taste  for  the  first time  but  then  cannot 
experience it again because of the wine being present there everytime. Thus, there is a clear indication to overabundance of wines, and hence option 
(3) is correct. 
24. Ans.(4). Ethereral as used in the passage can correspond to an experience which is out­of­ordinary, or simply something higher than earthly things. All 
options except option (4) refer to something that is out of ordinary. Option (4) erroneously links common­sense to ethereal. 
25. Ans.(2). Option (1) finds a mention in paragraph 4. Option (3) is there in paragraph 2 and option (4) is there in the next to last paragraph. Option (2) is 
incorrect as the passage states that Gamay grape was considered to be unfit for human consumption and hence was not grown. Pinot Noir was grown 
for making wine. Gamay grape was not the reason for growing Pinot Noir. 
26. Ans.(2).  The  argument  in  option  (2) is essentially  that  the  proposed  creation  of  a  Censor  Board  makes no  sense  because sitcoms  show  the  same 
content.  The  argument  relies  explicitly  on  an  analogy  between  movies  and  sitcoms.  Thus,  the  two  must  be  similar  in  all  respects  relevant  to  the 
argument. Otherwise, the argument is unconvincing. Option (2) affirms that sitcoms are in fact detrimental, thereby solidifying the analogy between the 
two. (1),  (3) and (4) each  in  its  own  way supports the bare assertion that the proposed Censor Board might not be effective. However, none of these 
answer choices confirms  the argument's essential reasoning.  (5) actually  weakens the argument, by  providing  a reason  why sitcoms and movies are 
not relevantly similar. 
27. Ans.(5). A typical weekend IPL match generates more revenue than a typical Art Exhibition. Hence, increasing the number of weekend IPL matches is 
the surest  way, among the five  choices, for Detro to maximize revenues, especially  if the number of days per year that the stadiums are used for Art 
Exhibitions  would  at  least  remain  the  same,  as  (5)  implies.  Options  (1)  and  (2)  are  incorrect  for  essentially  the  same  reason.  By  increasing  the 
admission  fee,  Detro  might  either  enhance or reduce  its  total  revenues, depending  on  the  decrease in  attendance  (if  any)  due  to  the  fee  increase. 
Besides, neither plan would add to the number of days during which one or the other type of event takes place. Hence, neither is as likely to succeed 
as the one that option (5) suggests.  (3) is the worst  of the five choices, and the  easiest  one to eliminate; replacing an event with one that  generates 
significantly  less revenue can only serve to reduce revenues. Option  (4) suggests a course of action that is unlikely to be as effective as the one that 
option  (5)  suggests.  Increasing  the  number  of  weekend  exhibitions  might  boost  revenues.  However,  since  a  weekend  art  exhibition  generates  less 
revenue than a weekend IPL match, the boost would be less than if the number of weekend IPL matches were increased instead. 
28. Ans.(1). The  argument relies  on  the unstated  assumption  that no other event  since  the fest  could have  caused  the  outbreak  instead.  Statement  (1) 
provides  some  evidence that the students who  have reported  Psoriasis symptoms in fact contracted it at least one  week ago.  Accordingly, (1) helps 
support the  claim  that  it  was the  swim  in  the  contaminated water during  the  trip  two weeks  ago  that  caused  the  outbreak. (4) has  no  effect  on  the 
argument. It is the time after contact that symptoms begin to occur, not the duration of those symptoms, that is key to identifying the source of spread. 
(2) actually weakens the claim, by providing another possible explanation for the outbreak. Specifically, (2) provides for the possibility that the outbreak 
can be attributed to the water in the swimming pool of the school rather than the dip at the picnic. (3) provides some support for the argument, insofar 
as it helps to explain why only one of those reporting symptoms has tested positive so far. Additional statistical information would be required to assess 
whether (5) supports the argument. For example, assume for the moment that  several  students­­including the 63 later reporting  Psoriasis symptoms­­ 
attended  the  trip.  In  this  event  (5)  would  strengthen  the  teacher's  claim,  by  providing  an  explanation  for  how  the  picnic  bath  could  have  been 
contaminated if only a small percentage of attendees have reported Psoriasis symptoms. Or assume on the other hand that the only students attending 
the trip were the 63 later reporting Psoriasis symptoms. In that case all who took a bath showed symptoms for Psoriasis and hence, teacher’s assertion 
will be repealed. 
29. Ans.(5). The argument relies on the unstated assumption that Tamil Nadu's Karanatakan population either remained stable or increased during 2007. 
However,  (5)  provides  that  this  population  actually  declined  in  2007,  despite  the  influx  of  Karanatakans.  Given  that  the  number  of  Karanatakans 
residing  in Tamil Nadu  decreased  while the  crime  rate  increased, (5)  reduces the  likelihood  that  it  was  Karanatakans  who  were  responsible  for  the 
increase in violent crime in 2007. (1) would appear to weaken the argument, by providing ostensible evidence that Tamil Nadu citizens are more likely 
than  Karanatakans  to  commit  violent  crimes.  However,  (1)  does  not  account  for  the  possibility  that  in  Karanataka  far  more  violent  criminals  go 
unapprehended than in Tamil Nadu. In fact, the argument's explicit reference to "reported" violent crimes underscores this possibility, which prohibits us 
from drawing any firm conclusion as to  which group  is more likely responsible  for violent crimes.  (2) provides  no  information  useful  in evaluating the 
argument. Whether (2) strengthens the argument depends on additional considerations as well, such as: the total population of Karanataka compared 
to Tamil  Nadu;  whether the  Karanatakan population  increased or decreased in each  state during the  year; and whether the  crime  rate in  Karanataka 
increased or decreased during 2007. (3) actually strengthens the argument. By providing evidence that number of Karanatakans residing in Tamil Nadu 
increased in 2007, (3) makes it more likely that Karanatakans were responsible for the increase in violent crime that year. (4) actually strengthens the 
argument, by affirming the essential premise that the number of violent crimes in Karanataka increased dramatically during 2007. 
30. Ans.(1). According to the passage, the total aluminium demand for packaging has been decreasing, while the amount of aluminium packaging recycled 
for  the  same  purpose  has  been  increasing.  These  two  facts,  considered  together,  lend  strong  support  to  the  conclusion  that  manufacturers  have 
substituted recycled aluminium­packaging for "fresh" aluminium to meet their demand for aluminium packaging. (2) is not strongly  inferrable from the 
passage, because without any numbers, it is impossible to compare the total amount of recycled packaging to the total amount of "fresh" packaging. (3) 
is incorrect because the passage provides no information for the inference that all recycled aluminium packaging has been used to meet the aluminium­ 
packaging demands of manufacturers. (4) is not a strong option. The only information in the passage about the demand for packaging material involves 
aluminium packaging. It is impossible to draw any strong conclusions about the demand for non­aluminium packaging material. In (5), just because the 
amount of recycled aluminium packaging has increased, it is unfair to conclude that the amount of aluminium from products has also increased. 
31. Ans.(2). It is clear that in protectionist policies one country would protect its own jobs whatever the type of job may be. Thus, global is incorrect. Since, 
the paragraph is not worried about the type of job, we can even cancel out options (3), (4) and (5). 
32. Ans.(1). Since we are talking of a grave situation, option (1) clearly is the best choice. Option (2) is in opposite context. Option (3) though might seem 
true with respect to protectionism but does not fit in because nowhere has the authenticity of situation been questioned. Option (4) is not right because 
such a situation already occurred in 1930. Option (5) is close but ponderous (heavy; or having a great mass and weight and unwieldiness) is not of the 
required level that is expected from an occurence that causes end of globalization.

IC : PTpnrps03  3 of (17) 
33. Ans.(4). Selfish  in this option  cannot fit because  it would be  redundant to  speak "selfish protectionist" when protectionism has the meaning of  selfish 
built  in. Tender will  be  inept  because  the  speaker is condemning protectionism.  Fearsome  implies  something  that  is dreadful  or  frightening.  Fearful 
might have been an apt  choice. Mediocre  doesn't explain anything. Only short­sighted  seems correct as the speaker warns that  decisions like  these 
may hamper growth in the future. 
34. Ans.(3). Since we are talking of a threat against globalization, options (2) and  (4) are  ruled out. Options (1) and (5) talk  of temporary hindrance,  but 
option (3) says choke ­off; i.e. ,complete devastation and thus option (3) most aptly defines the threat. 
35. Ans.(2). Since we are talking of resisting protectionism, (1) and (5) are rejected as instead of tackiling it, they would further promote it. Options (3) and 
(4) are general words and do not fully bring out the graveness of the need to work together. Hence option (2) is the right answer. 
36. Ans.(3). Options (1) and (4) that mention "special mark" presupposing that it has been introduced are incorrect as it  is nowhere in the first part of the 
paragraph. Option (5) is close but is incorrect because had it been in the paragraph, the  construction would have looked like, "Its not just this attitude 
that is mysterious. Its also their special mark". There is an also missing there. Option (2) is wrong because long eyelashes and sombre face does not 
prepare the donkeys for long journeys. Only option (3) gives a link between the two parts. 
37. Ans.(2). A clear look at the lines above the missing one will reveal that what we need in the missing line is something that shows what the Government 
can do (using its buying  power) to improve the  security of the products. Thus,  options  (1),  (3), (4) and (5)  all get negated. Option  (1) speaks of fake 
products  (an  unnecessary  inclusion).  (4)  specifies  that  the  cybersecurity  issue  will  be  dealt  with  in  the  fullest,  while  the  statement  in  the  last  line 
mentions "improved security" not "complete security". (3) and (5) talk nothing about the "buying power" of the Government. 
38. Ans.(3).  A  careful  analysis  will  reveal  that the author  is trying to hold  the  American  leadership  and  the  citizens  responsible  for  the  sorry  state  that 
America  portrays  on  the  global  front  today.  Thus,  options  (1),  (4)  and  (5)  can  be  negated  because  they  are  trying  to  say  that  the  world  has  been 
unworthy of America's efforts. Among options (2) and (3), option (3) is a brilliant filler because it quickly ditributes the onus of the responsibility for the 
American predicament between the people and the leaders. Option (2) could have been a better choice if only option (3) was not there. 
39. Ans.(4). This one should be easy. When one compares the options and looks up what are the differences, she/he can easily make out two: indicate(s); 
and  lie/lay. Considering the  subject­verb  agreement, "information"  will  entail  an  "indicate".  Thus,  options (1),  (2)  and (3) are  ruled  out.  Option  (5)  is 
wrong because it uses the incorrect verb "lay" rather than "lie". Lay usually implies "put into a certain place or abstract location". While lie is "to have a 
place in relation to something else". 
40. Ans.(1).  Here  we  need to  be  clear about  the usage of few,  fewer,  less and  lesser.  'Fewer'  answers the  question,  "How  many?"  (bats,  balls,  dogs, 
chairs)  and  'less'  answers the  question,  "How  much?"  (cola,  dirt,  salt,  capacity).  Fewer  and  lesser  are  always  used  when  comparing  things.  This 
eliminates answer choices (2), (4), and (5). Answer choice (3) is unnecessarily wordy and implies that the auditoriums had already been built before the 
buttonholing began. 
41. Ans.(3). Statement A can be classified as an inference because even though we do not have the data from where this conclusion has been drawn, yet 
we are sure that it is a conclusion atleast.  B is a judgement  because of the use of the  word "narrowly". This is an  unquantifiable adjective,  i.e. it can 
change according to the perspective of the person and the tendency of the situation. C and D are facts and can be easily verified. 
42. Ans.(5).  Statements  A  and  C are  facts that  can  be  easily  verified.  B  is acknowledged  as  a  fact  because  it  is  something  that  someone  has  spoken 
publicly. This can be verified for its truth value. D is an inference because a conclusion has been drawn in the later part of the sentence based on the 
former half. 
43. Ans.(2). Statement A is an inference drawn from data that is not provided in the question. B and C are inferences based on information given in A and 
B respectively. D is a fact because the function of the State Water Resource Regulatory Authority can be confirmed. 
44. Ans.(1). Statement  A is a fact and can be  verified whether differnet  cases of  incest  have come  up  in the  country or not after the expose of the Mira 
Road case. C is a fact and can be identified first among all. B is a conclusion drawn based on the information provided in A. D is a judgement becasue 
of the unquantifiable phrase "frowned upon". One cannot say for certain what this may mean to the speaker. Hence, option (1) is the correct choice. 
45. Ans.(4). Statement A is a judgement of the situation presented by the speaker. Statements B and C are conclusions drawn. In B, we can see "How that 
will pan out  remains uncertain"  is the  conclusion drawn on the basis of  information provided ahead  in the  line. In C, the  second part  of the  sentence 
after the comma is the inference drawn as the speaker clears that the "No Means Yes" brigade may have to move on. D is a clear judgement that the 
speaker is passing on the prevalent situation. 
46. Ans.(3). The quote means that mercy blesses both the giver and the taker and its blessings are not exhausted by the increase in the number. That is 
why, everyone should show mercy to others. 
47. Ans.(4). An optimist's quest for  looking at the brighter side everywhere can  lead to trying situations.  Any person who gets through these traumas  can 
be pardoned if he doubts things and becomes a pessimist. Option (4) is the best answer. Option (3) is near but it labels all optimism as bad which is not 
what is meant here. 
48. Ans.(4). The quote implies that real intelligence is comprehending the length and the breadth of one's knowledge. Only option (4) is correct. 
49. Ans.(4).  The  quote  can be  understood  as "a  custom  that  is honoured  by  not  observing it  and  rather breaching  or avoiding  it."  Hence,  option  (4)  is 
correct. 
50. Ans.(3). The opposite of a true  statement  can be a false  statement.  But as the quote says the opposite  of a profound truth can be another profound 
truth. This implies that truly nature doesn't distinguish between right or wrong and treats both of them equally. Thus, option (3) is the only correct option.

4 of 17  PTpnrps03 
For Questions 51­54 
The number of people in each of these categories can be calculated by adding all the values of the columns: 
3(Lords + Vyaparies + Brahmans + Kshatriyas + Vaishya + Shudra + Bhikshuk) = 3375 
Hence, total number of people from all the categories =1125 
Now, the number of people from the Shudra category = 
1125 –(Brahmans +Kshatriyas + Vaishya) – (Bhikshuk + Lords + Vyaparies) 
= 1125 – (425) – (600) = 100 
Similarly, all the values can be determined: ­ 

Lords  250 

Vyaparies  200 

Brahmans  75 

Kshatriyas  50 

Vaishya  300 

Shudra  100 

Bhikshuk  150 

51.  For getting the minimum number of categories that have more than 50 people unhappy, 
70% of 50 = 35 
55% of 75 = 41.25 
33% of 100 = 33 
30% of 150 = 45 
24% of 200 = 48 
20% of 250 = 50 
65% of 300 = 195 
Hence, only 1 category is possible. Ans.(1) 
52.  The number of categories with more than 200 people can be counted from the above table (Lords and Vaishyas). Ans.(5) 
53.  The number of unhappy people in the categories, Vaishya, Lords, Vyaparies, Bhikshuk, Shudra and Brahmans can be at the most more than 40 if the 
percentage distribution is: 20, 24, 30, 33, 55, 65 and 70 respectively. Ans.(4) 
54.  If  the  number of  people  in  the  Shudra  category  is  reduced  by  25%,  then  the  number  of  people  =  100  ×  75%  =  75.  70%  of  75  =  52.5.  Thus,  the 
maximum number of unhappy people = 52. Ans.(5) 
55.  It is clear that if we know who made the sixth move, we also know who can make the seventh move and can eventually win. 
If Gautam (zeroes) has made the sixth move, there are three possible situations after five moves: 
Possibility 1: Possibility 2: Possibility 3: 
O  O  O  O 
O  X  O  X  X 
X  X  X  X  X  X
Based on the rules of the game, only possibility 1 could have resulted in the present situation after six moves. In that case, there are three possibilities 
for ‘possibility 1’ to result in the present situation 
Thus, after four moves: 
Possibility 1: Possibility 2: Possibility 3: 
O  O  O 
O  X  O  O  X 
X  X  X  X
Based on the rules of the game, Maithli (crosses) would have made the winning move, which however did not happen. From this we can conclude that 
Maithli  did  not  make the  fifth  move  and hence,  Gautam  did  not  make  the  sixth  move.  So,  it  must have  been  Maithli  who  made  the  sixth  move  and 
Gautam then makes the seventh, winning move. 
To check that Maithli made the sixth move, we look at the the following three possibilities after five moves: 
Possibility 1: Possibility 2: Possibility 3: 
O  O  O  O  O  O 
O  O  X  O  X 
X  X  X  X
Based on the rules of the game, only possibility 3 can result in the situation after six moves. So, Maithli indeed made the sixth move. Ans.(2)

IC : PTpnrps03  5 of (17) 
For Questions 56­57 
We will indicate the numbers in the long division in the manner shown below: 
_ _ _ / _ _ _ _  _ _  _ _ \ _ _ 8 _ _  A / B \ C 
_ _ _  D 
­­­­­­­  ­­­ 
_ _  _ _  E 
_ _ _  F 
­­­­­­­­­  ­­­ 
_ _  _ _  G 
_ _  _ _  H 
­­­­­­­­­  ­­­ 
0  0 
For E and G, two blank spaces have been brought down; therefore both the second and the fourth space of C are 0. 
Because F consists of three  spaces and  equals the  outcome of 8 times  A,  A  must be smaller than 1000/8 or 125.  Number H  consists of four  spaces 
and is the product of number A and the last space of C. Therefore, H is at least 1000, and because A is smaller than 125, the last space of C must be 
9. Moreover, A must be at least 1000/9  and hence, larger than 111. 
Because the number D is subtracted from a number represented by four  spaces and the outcome of the subtraction  consists of two spaces, the first 
space of D must be 9. This is possible only if the first space of C is 8. 
Now we know that the number C equals 80809 and number B is at least 10000000. From this, it follows that A is at least 124. Because we also know 
that A lies between 111 and 125, A equals 124. 
The complete long division can now be filled in: 
1 2 4 / 1 0 0 2 0 3 1 6 \ 8 0 8 0 9 
9 9 2 
­­­­­­­ 
1 0 0 3 
9 9 2 
­­­­­­­­­­ 
1 1 1 6 
1 1 1 6 
­­­­­­­­­­ 

56.  The problem has a unique solution. Ans.(1) 
57.  From 0 to 9, only 5 and 7 are not required to complete the long division. Ans.(2) 
For Q.58 to Q.61 
58.  To find the maximum number of employees who were recruited in January and not fired at the end of August, the fired employees must include all the 
employees of that particular month except January and if there are more number of fired employees in a month than the recruited ones, only then we 
will assume the January employees to be fired.
TABLE­1
New recruitments(at the Employees who were
Fired Emoloyees(after the
Month beginning of the not fired since the
corresponding month)
corresponding month) month of January 
January  80  55  25 
February  70  74  25­4=21 
March  45  48  21­3=18 
April  56  66  18­10=8 
May  80  70  8 
June  62  73  8­(11­10) 
July  56  50  7 
August  20  6  7 
Hence, 7 employees were recruited in January and were not fired till the end of August. Ans.(3)

6 of 17  PTpnrps03 
59.  To find the maximum number of employees who were recruited in March and were not fired till the end of June, we need to follow the table:
Fired Employees Employees recruited
New recruitments(at the
(after the in March and not fired
Month beginning of the
corresponding at the end of the
corresponding month)
month) month 
January  80  55 
February  70  74 
March  45  48  18 
April  56  66  8 
May  80  70  8 
June  62  73  7 
July  56  50 
August  20  6
Ans.(3) 
60.  The employees who were recruited in a particular month and fired exactly after 3 months would be the employees who were recruited in the months of 
January, March and May. 
So, from table 1, the number of employees who were recruited in January and were fired at the end of March were 21. 
The so the number of employees who were recruited in the month of March and were not fired in the same month were 18 as shown in the table. In the 
month of April 10 employees from the previous month were fired. Thus, the number of new recruitments in March who were not fired in  the next month 
were 8. 
In the month of May 80 employees were recruited at the begining and 70  were fired  at the end of the month.  So, 8 employees who were recruited in 
March were fired at the end of May while 18 employees who were recruited in May were not fired at the end of same month. In the month of June, total 
73 emplyees were fired out of which 62 must be new June recruitments. Thus, 11 employees who were recruited in the month of May were fired at the 
end of June. So, 7 emplyees who were recruited in May were left and were fired at the end of July. So, maximum total number of employees who wre 
recruited in a month and  were fired after exactly after three months were 21  + 8 + 7  = 36. Hence, the  total amount the  company  would  have paid  to 
these employees = 36 × 15000 = 540000 Rs. Ans.(3) 
61.  The maximum number of employees who were recruited at the beginning of a month and fired at the end of the same month were 55 + 70 + 45 + 56 + 
70 + 62 + 50 + 6 = 414. Ans.(3) 
For Q.62 to Q.64 
Given that 40% of the total employees who joined AMA as Marketing Executives = 21720 
So, total employees from the other company who joined AMA = (21720 × 100)/40 = 54300 
This is equal to 80% of the total employee base of the other company. 
So, the total number of employees of the other company = (54300 × 100)/80 = 67875 

Total number of 
80% of  16% of 80% of OC  44% of 80% of OC  40% of 80% of OC 
employees of other 
OC  who joined HR  who joined Finance  who joined Marketing 
company (OC) 
67875  54300  8688  23892  21720 
16% of the total employees who joined as HR executive AMA = 20% of the total employees who failed the promotion test = 8688 
So, total employees who failed the promotion test = (8688 × 100)/20 = 43440 
This is equal to 30% of the total employees of AMA. 
So, total number of employees of AMA before the merger = (43440 × 100) / 30 = 144800 
Total number of employees who passed the promotion test = (70 × 144800) / 100 = 101360 

30% of TEP 
Total employees who  25% of TEP promoted as  40% of TEP promoted as 
promoted as Finance 
passed the test (TEP)  HR executive  Marketing executive 
executive 
101360  25340  30408  45612 
62.  So the total number of employees after the merger = 144800 + 54300 = 199100. Ans.(1) 
63.  We can calculate only the number of male employees before the merger.  But  we cannot calculate the number of male or female employees after the 
merger, because we do not know about the number of males or females who joined AMA from the other company. Ans.(5) 
64.  Following table is prepared from the given information:
Total number Other
Company HR Marketing Finance
of employees employees 
AMA  144800  25340  45612  30408  43440 
OC  67875  8688  21720  23892 
212675  34028  67332  54300  43440 
Old Finance executives = 30408 
25% got pink slip, so, remaining 75% = (75 × 30408) / 100 = 22806 
15% of the HR executives changed their department and joined as Finance executives, i.e. (15 × 34028)/100 = 4563 
So, total Finance executives = 4563 + 22806 + 23892 = 51261. Ans.(3)

IC : PTpnrps03  7 of (17) 
For Questions 65­68 
From the given information, following tables can be prepared:
Table I
Price in INR
Dress (1 Euro = 65 (1 USD = 50 (1 SGD = 35
(1 GBP = 75 INR) (1 AUD = 30 INR)
INR) INR) INR)
Ladies Evening Dress  65000  135000  60000  60000  45500
Knit Cover up  91000  58500  84000  49500  68250
Voile Cover up  58500  84750  65000  56100  57750
Cotton Ladies Tunic  103350  69000  72500  63000  68250
Embroidered Tunic  78000  125250  88000  54000  50750
Stylish Ladies Tunic  88400  115500  71500  35400  65100
Party wear saree  77350  71250  43000  52500  44100
Wedding Wear  149500  197250  114500  101400  97300

Table II 
Custom Duty on goods

Dress bought from Milan is  bought from London  bought from San  bought from  bought from 


1%  is 2%  francisco is 0%  singapore is 20%  Rosemount is 5%

Ladies Evening Dress  650  2700  0  12000  2275

Knit Cover up  910  1170  0  9900  3412.5


Voile Cover up  585  1695  0  11220  2887.5

Cotton Ladies Tunic  1033.5  1380  0  12600  3412.5

Embroidered Tunic  780  2505  0  10800  2537.5

Stylish Ladies Tunic  884  2310  0  7080  3255

Party wear saree  773.5  1425  0  10500  2205


Wedding Wear  1495  3945  0  20280  4865
Table III

Total cost price (Basic price + Custom duty)


Dress  San francisco Singapore Rosemount
Milan (Italy) London (UK)
(USA) (Singapore) (Australia)
Ladies Evening Dress  65650  137700  60000  72000  47775
Knit Cover up  91910  59670  84000  59400  71662.5
Voile Cover up  59085  86445  65000  67320  60637.5
Cotton Ladies Tunic  104383.5  70380  72500  75600  71662.5
Embroidered Tunic  78780  127755  88000  64800  53287.5
Stylish Ladies Tunic  89284  117810  71500  42480  68355
Party wear saree  78123.5  72675  43000  63000  46305
Wedding Wear  150995  201195  114500  121680  102165
65.  From table III, the minimum amount that Mrs. Aadhunikwala must carry = 2 × 70380 = 140760 (from London Fashion Week, UK) Ans.(5) 
66.  From  table  III,  the  price  of  Ladies  Evening  Dress  at  London  Fashion  Week  (UK)  =  137700  and  the  price  of  Ladies  Evening  Dress at  Rosemount 
(Australia) = 47775. So, the maximum difference = 137700 ­ 47775 = 89925 Ans.(2) 
67.  We can calculate the price of all these five dresses from table III 
Party Wear Saree, Rosemount Fashion Week = 95% of 46305 
Voile Cover Up, Milan Fashion Week = 95% of 59085 
Ladies Evening Dress, Rosemount Fashion Week = 95% of 47775 
Party Wear Saree, San Francisco Fashion Week = 95% of 43000 
Stylish Ladies Tunic, Singapore Fashion Week = 95% of 42480 
So, we can observe that Stylish Ladies Tunic at Singapore Fashion Week is the cheapest. Ans.(5)

8 of 17  PTpnrps03 
68.  Ladies Evening Dress, Rosemount Fashion Week =   47775 
Party Wear Saree, San Francisco Fashion Week = 43000 
Stylish Ladies Tunic, Singapore Fashion Week = 42480 
So, the maximum number of dresses that she can buy with an amount of Rs.180000 (INR) is 3. Ans.(1) 
For Questions 69­71
Number of programs offered by University 
Year of Nanda Sherpa
Taxla University Appa University Rana University
Inception University University
2001  9  6  12  11  20
2002  12  18  8  7  17
2003  6  17  13  16  11
2004  17  9  17  10  18
2005  6  13  14  11  6
2006  19  15  9  13  12
2007  18  16  10  5  19
2008  13  11  18  19  7
Total  100  105  101  92  110
Average 12.5 13.125 12.625 11.5 13.75
69.  There are 4 universities which on an average offer more than 12 programs. Ans.(4) 
70.  There are three universities which offer more than 100 programs.
Number of programs offered by University 
Year of Taxla Appa Rana
Inception University University University
2001  6  12  20
2002  18  8  17
2003  17  13  11
2004  9  17  18
2005  13  14  6
2006  15  9  12
2007  16  10  19
2008  11  18  7
Total 105 101 110
P1 = 105 + 101 + 110 = 316 
P2 = 100 + 105 + 101 + 92 + 110 = 508 
Hence, required percentage = (316 × 100)/508 = 62.20 % Ans.(2) 
71.  The maximum percentage drop in new programs offered was by Rana University in the year 2005 over 2004. Ans.(5) 
For Questions 72­75 
72.  From the question we get the following table ­ 
R a ting 
WH A /  T REAT ME 
Ho sp ita l n a me  Me dic in a l  In fra struc tu re 
WO HA  N T US ED 
re se rve  &  e qu ipme n t 
CHILDREN'S 
WOHA  ART, AT  7.5  7.5 
HOSPITAL 

CIVIC CENTER 
HOSPITAL  WOHA  UT, AT  9  6 
FOUNDATION 
ESKATON DOCTORS 
WOHA  HT, MT  9  9 
HOSPITAL 
HAYWARD HOSPITAL  WOHA  UT, AT  13.5  10.5 

NAVAL HOSPITAL  WOHA  ART, AT  10.5  10.5 

OGORMAN INFANT  WOHA  AT, HT  9  6 

PERALTA HOSPITAL  WOHA  HT, MT, UT  7.5  4.5 

Hence, there are 7 hospitals where Mr. Chopar can get treated. Ans.(3)

IC : PTpnrps03  9 of (17) 
73.  From the question we get the following table: 

Ra tin g 

T REAT M  S um o f  R a tio of ra ting 


H ospita l  WH A /  Me d ic in  ra tin g o f  o f fe e d  b a c k to 
ENT 
na me  WO HA  a l  Do c to rs  F e e db a c k  me dic in a l  su m o f ra ting 
US ED 
re se rve  re se rve  of me dic in a l 
a nd  d oc tor  re se rve   a n d 
do c tor 
BOOTH 
HT, ART, 
MEMORIAL  WHA  10.5  9  10.5  19.5  0.538461538 
RT 
HOSPITAL 

CHILDREN'S 
WOHA  ART, AT  7.5  10.5  13.5  18  0.75 
HOSPITAL 
SAN 
MT, ART, 
LEANDRO  WHA  7.5  7.5  10.5  15  0.7 
UT 
HOSPITAL 
EDEN 
MEDICAL  WHA  HT, UT  7.5  7.5  7.5  15  0.5 
CENTER 
SUMMIT 
RT, MT, 
MEDICAL  WHA  10.5  13.5  12  24  0.5 
ART 
CENTER 
OAKLAND 
AT, ART, 
HOSPITAL  WHA  10.5  9  10.5  19.5  0.538461538 
UT 
CORPORATI 
OGORMAN 
WOHA  AT, HT,  9  9  9  18  0.5 
INFANT 

PERALTA 
WOHA  HT, MT, UT  7.5  7.5  7.5  15  0.5 
HOSPITAL 

SUMMIT 
RT, ART, 
MEDICAL  WHA  9  9  12  18  0.666666667 
UT 
FOUNDATION 
HERRICK 
MEMORIAL  WHA  RT, UT  13.5  9  10.5  22.5  0.466666667 
HOSPITAL 
Hence, there are 9 hospitals. Ans.(5)

10 of 17  PTpnrps03 
74.  Number of hospitals which do not have In House facility of Ambulance having ART or HT and their rating of doctors is more than 7.5 are given below. 
WHA /  TREATMENT 
CHILDREN'S HOSPITAL  Doc tors 
WOHA  USED 
CHILDREN'S HOSPITAL  WOHA  ART, AT  10.5 
NAVAL HOSPITAL  WOHA  ART, AT  9 
OGORMAN INFANT  WOHA  AT, HT  9 

Also, number of hospitals which have In House facility of Ambulance having RT or UT and their rating of Infrastructure & equipments equal or greater 
than 7.5 are given below. 

WH A /  T REAT ME  In fra stru c ture  & 


Ho sp ita l na me 
WO HA  N T US ED  e q uip me n t 

BOOTH MEMORIAL 
WHA  HT, ART, RT  9 
HOSPITAL 

SAN LEANDRO 
WHA  MT, ART, UT  7.5 
HOSPITAL 

HERRICK MEMORIAL 
WHA  RT, UT  9 
HOSPITAL 

KAISER HOSPITAL  WHA  UT, RT, HT  12 

KAISER HOSPITAL  WHA  AT, UT  15 

SUMMIT MEDICAL 
WHA  RT, MT, ART  10.5 
CENTER 

OAKLAND HOSPITAL 
WHA  AT, ART, UT  10.5 
CORPORATION 

SUMMIT MEDICAL 
WHA  RT, ART, UT  7.5 
FOUNDATION 

Hence, the required ratio = 3/8. Ans.(1) 


75.  From the inforamtion in the question, only following three hospitals have both RT and ART treatment facilities 
OVREALL 
Rating
RATING 

{(Rating of 
WHA / 
Hospital  TREATME  RATING OF  feedback) × 
WOH  Medic i  RATING 
name  NT USED  feedb  MEDICINAL  5 + 3 
A  nal  OF FEED 
ac k  RESERVE ×  ×(Rating of 
reserve  BACK × 5 
3  medic inal 
reserve ) } 

BOOTH 
HT, ART, 
MEMORIAL  WHA  10.5  10.5  52.5  31.5  84 
RT 
HOSPITAL 
SUMMIT 
RT, MT, 
MEDICAL  WHA  10.5  12  60  31.5  91.5 
ART 
CENTER 
SUMMIT 
MEDICAL  RT, ART, 
WHA  9  12  60  27  87 
FOUNDATIO  UT 

Ans.(2) 
76.  Factorizing the numbers, 
120 = 2 3  x 3 1  x 5 1 
1320 = 2 3  x 3 1  x 5 1  x 11 
A is definitely a multiple of 11, but it cannot be a perfect square as A cannot have 11 2  as a factor. 
1680 = 120 x 2 x 7 = 2 4  x 3 1  x 5 1  x 7 1 
B is definitely a multiple of 2 4  x 7 1  but it cannot be a perfect square as B cannot have 7 2  as a factor. 
1800 = 120 x 3 x 5 = 2 3  x 3 2  x 5 2 
Since, the LCM of 120 and C is 1800, therefore, C can be a perfect square. One of the possible values of C is 3 2  x 5 2 . But it is not necessary that C be 
a perfect square. Ans.(5)

IC : PTpnrps03  11 of (17) 
77.  Since, in the new number system, decimal number 5 is represented by a single sign, so the base of that number system is atleast 6. 
Similarly, the decimal number 95 is represented by three signs, so the base of this number system is not more than 9. 
Now,  checking  for  base  6, decimal  number 20  can  be  written as (32)6  and  decimal  number  95  can  be  written  as  (235)6.  But  each  sign represents 
different decimal number. So, base 6 is not possible. 
Now,  checking for base 7, decimal number 5  is  represented by (5)7, decimal number 20  can be represented by (26)7  and decimal number 95  can  be 
represented by (164)7. So, according to the signs provided in the question, 1 = 'µ', 2 = '+', 4 = '#', 5 = '$' and 6 = '%'. 
Therefore, decimal number 778 can be written as (2161)7  in the number system of base 7 
Or, (+µ%µ). Ans.(4) 
78.  (x + 1)yz = 12  ………………(1) 
(y + 1)zx = 4  ………………(2) 
(z + 1)xy = 4  ………………(3) 
Dividing equation (1) by (2), 
(x  +  1)y 
= 3 
(y  +  1)x 


x = 
(2y  +  3) 

Dividing equation (2) by (3), 
z = y 
Now, on subtracting equation (2) from equation (1), we get 
y 2 – xy = 8  (Since, y = z) 

x = 
(2y  +  3) 
by putting the value  , the equation becomes 

2  y 2 
y  - = 8 
(2y  +  3) 
On solving for y, we get 
y = –2, 3 
Now, for y = –2, xyz = 2 × –2 × –2 = 8, an integer. 

for y = 3, xyz =  3  × 3 × 3 = 3, an integer.
Ans. (5) 
79.  Given:  A 5  = B 4 , also A, B, C and D are positive integers. Hence, only possible values of A and B are A = 1 and B = 1. 
Now, (C – A) = 3, so C = 4. 
Also, C 6  = D 2 
Hence, 4 6  = D 2 
So, D 2  = 4096, so D = 64. 
Therefore, D – A = 64 – 1 = 63. Ans. (4) 
80.  Let the speed of the bus and the scooter be x and y respectively, 
Distance from A to Mumbai = 2x 
Distance from Mumbai to B = x/2 
Distance from A to B = 2x ­ x/2 = 3x/2 
To cover a distance 3x/2 (from A to B), the scooter takes 7/2 hours (from 2pm to 5:30 pm) 
Now,  to  cover the  remaining  distance  from  B  to  Mumbai  i.e.  x/2, the  scooter  will take  7/6  more  hours  or 1 hour 10  minutes  from 5:30  pm.  So,  the 
scooter will reach Mumbai at 6:40 pm. Ans.(2) 
81. Let A be the event that there are exactly 4 Chinese among the first 6 speakers.
Let B be the event that the seventh speaker is a Chinese. 
F G B I J 
Then, Required Probability = P(A) ´

H A K 
Now, 

C 4 ´ 7 C 2  F G B I J  1 
P(A) = 
12 
C 6 
and 
H A K =Prob. that the seventh speaker is a Chinese given that there were 4 Chinese speakers among the first 6 speakers=  6 


C 4 ´ 7 C 2  1  5
12 
´ 
C 6  6 
Hence, required probability =  =  264 Ans.(3)

12 of 17  PTpnrps03 
82.  On generalizing the above equation, 
F(x)=2x 2  + a(b–c) 2 x + abc 
We know that for an equation ax 2 +bx+c=0, 
the minimum or the maximum value of f(x) lies at x = –b/2a. 
Therefore, –a[(b–c) 2 ]/4 = –128 or a(b–c)=512 
Now, the sum a + (b­c) + (b­c) + (b­c) will be minimum when 
a = b–c = 8 
Hence, the minimum value of a + 3(b­c) = 8 + 8 + 8 + 8 = 32 Ans.(3) 
83.  1st process ­ 
Divide the larger square in 4 equal squares of side A/2. Then the area of total scrap 
= 4[(A/2) 2  – p(A/4) 2 ] = A 2 (4 ­ p)/4 
2nd process ­ 
Area of total scrap generated in this process = A 2 – p(A/2) 2  = A 2 (4 – p)/4 
So, required ratio = 1:1. Ans.(3) 
84.  For a given volume, cuboidal shape of minimum surface area is a cube. So, he will arrange all the cubes in the form of a large cube and the side of this 

cube will be 9 cm. Therefore, the length of the diagonal =  9  3  cm. Ans.(4) 


85.  Let the number be n = a p  × b q  × c r ……………… 
where a,b,c are prime numbers & p,q,r natural numbers. 
We know numbers of facters of n = (p+1)(q+1)(r+1)…………= 4 
So, either 1×4 or 2×2 = 4 
Or, p+1 = 4 & q = 0 or p+1 = q+1 = 2 
p = 3 & q = 0 OR p = q = 1 
either n = a 3  or n = a×b 
So, a + a 2  = 72, i.e., a = 8 and n = 512 
Or, a + b = 72 assuming a<b 
(a,b) = (5,67), (11,61), (13,59), (19,53), (29,43), (31,41) 
Hence, required sum = 512 + 5×67 + 11×61 + 13×59 + 19×53 + 29×43 + 31×41 
= 512 + 335 + 671 + 767 + 1007 + 1247 + 1271 
= 5810. Ans.(4) 

A  B 

86.  Q  S  R 
The sides of triangle PQR = 1unit 

PO = OS =  3  / 4 units and angle AOP is 120° (Central angle theorem) 

1  3  3  1  3  3  3  3  3 
´ ´ ´ Sin 120 ° = ´ ´ ´ = 
Area of triangle POA = Area of triangle POB =  2  4  4  2  4  4  2  64 

F  3  I  = p 

120 °
360 °
´ p G H  4  JK  16 
Area of the minor sector OASB = 
Area  of  the  part  of  triangle  that  lies  outside  =  Area  of  triangle  PQR  ­  [Area  of  triangle  POA  +  Area  of  triangle  POB  +  Area  of  minor  arc  OASB] 
3  / 4 - [ 3  3  / 32 + p / 16 ] = ( 5  3  - 2 p ) / 32 
=  . Ans.(3) 
87.  If (a,b) is a solution then obviously a = 0 and (a,­b) is a solution too. For a = 0, we get two solutions (0, 2) and (0, ­2). 
Now, let (a,b) be a solution with a>0; 
The equation can be rewritten as, 
2 a (1+2 a+1 ) = (b+1)(b–1) 
Shows that the factors b–1 and b+1 are even, exactly one of them divisible by 4. Hence a ³ 3 and one of these factors is divisible by 2 a­1  but not by 2 a . 
So, b = 2 a–1 m + c­1, m odd  ………………….(1) 
Putting this into the original equation we obtain 
2 a (1 + 2 a+1 ) = (2 a­1 m + c–1) 2 –1=2 2a–2 m 2 +2 a mc 
or, equivalently

IC : PTpnrps03  13 of (17) 
1+2 a+1  = 2 a–2  m 2  + mc. 
Therefore, 1– cm = 2 a–2 (m 2  – 8).  ………………….(2) 
For c = 1 this yields m 2  – 8 £ 0, i.e., m = 1, which fails to satisfy (2). 
For c = –1 equation (2) gives us 
1 + m = 2 a–2 (m 2  – 8) ³ 2(m 2  – 8), 
Or, 2m 2  – m – 17 £ 0. 
Hence, m = 3; on the other hand m cannot be 1 according to (2). Because m is odd, we obtain m = 3, leading to a = 4. From (1) we get b = 23. These 
values satisfy the given equation. Thus, (a,b): (0, 2), (0,­2), (4, 23), (4,­23). Ans.(3) 
88.  Let  a, b be the roots of the equation. 
The equation will have both the roots greater than 3 if it satisfies the conditions given below 
Condtion 1: D  0 
Condtion 2: a + b > 6 
Condtion 3: Value of equation at x = 3 is greater than 0 
From condition 1, 36a 2  – 8 + 8a – 36a 2  0  Or, a  1 
From condition 2, 6a > 6  Or, a > 1 
From condition 3, 9 – 18a + 2 – 2a + 9a 2  > 0 
Or, 9a 2  – 20a + 11 > 0 
So, a < 1 or a > 11/9 
Combining results from the above conditions, a > 11/9 satisfies all constraints. Ans. (3) 
37  x  17 
< < 
197  y  77 
89.  We have 
77 y  197 
< < 
or  17  x  37 

4.53 < £ 5.32 
or,  x 
x has minimum value 1 (it is a positive integer). To satisfy the given inequalities y can have a minimum value of 5 since it is also an integer. Ans.(3) 
90.  3 2  > 2 3 
2 log 2  3  > 3 log 2  2 
or 
log 2  3  > 3 / 2 
..........................(1) 
5 2  < 3 3 
2 log 3  5 < 3 log 3  3 

log 3  5  < 3 / 2 
..........................(2) 
9 > 8 
or 9/4 >2 
3 / 2  >  2  ..........................(3) 
and, 27>24 
(3/2) 3  > 3 

( 3 / 2 ) > 3  3 
..........................(4) 
log 2  3 
By comparing all the above four equations we can conclude that  is the largest number the given four numbers. Ans.(4) 
91.  The new hexagon formed, will have its side  equal to  Ö3/2 times the earlier hexagon. 
Thus, the perimeter will also be Ö3/2 times that of earlier Hexagon. 
Hence the perimeter of all the Hexagons thus formed will be sum of an infinite GP. 
= 540 / (1–Ö3/2). 
Or, 1080 / (2–Ö3). Ans.(4) 
92.  SE : ER = 5 : 4 
RF : FS = 8 : 1  P  Q 
SE = 5x and ER = 4x 
RF = 8y and FS = y 
so, SR = 9x = 9y 
or, x = y 
Let x  = 1  S  F  E  R 
Area of Trapezium PQFS = 1/2 (FS + PQ) × PS = 49

14 of 17  PTpnrps03 
On putting the values of FS and PQ, we get 
PS = 49/5 
Area of Trapezium PQRE = 1/2 (ER + PQ) × PS 
On putting the values of ER, PQ and PS, we get 
[1/2 × (4 + 9)] × 49/5 = 63.7 
Even if the value of x changes, the answer remains unchanged because the value of PS varies with x. Ans. (3) 
93.  The number of ways of choosing the three digits other than 0 and 1 = 8 C3  = 56 
These can be arranged in 6!/(2! × 2! × 2!) = 90 ways 
And the digit 1 can be placed in any of the seven possible locations with respect to the six digits. 
Total number of ways = 7 × 90 × 56 = 35280. Ans.(1) 
94.  log 10567 = log 10(81×7) = log10(3 4 ×7) = x 
4log 103 + log 107 = x 
log 101029 = log10(3×7 3 ) = log103 + 3log107 = y 
4log 103 + log 107 = x  ……………(1) 
log 103 + 3log 107 = y  ……………(2) 
By solving (1) and (2) 
log 107 = (4y–x)/11 
And 
log 103 = (3x–y)/11 
Now, log1021 = log10(3×7) = log103 + log 107 
=(4y–x)/11+(3x–y)/11 
=(3y+2x)/11 Ans.(1) 
95.  For getting a sum of 109 
The minimum number of cards required are 
(13 × 4 + 12 × 4 + 9 × 1) = 109, i.e., 9 
Similarly, (11 × 4 + 10 × 1 + 8 × 2) = 70, i.e., 7 
and, (10 × 3 + 9 × 2 + 8 × 2) = 64, i.e., 7. Ans.(1) 
96.  X = 20!(1/20! + 1/19! + 1/18!2! + 1/17!3! + … + 1/2!18! + 1/19! + 1/20!) 
Or, X = (20!/20! + 20!/19! + 20!/18!2! + 20!/17!3! + … + 20!/2!18! + 20!/19! + 20!/20!) 
These terms are binomial coefficients in the expansion of (1 + x) 20  where x is assumed to be 1 which will lead to the conclusion that 
20 C 20 C 20 C 20  20 
0  +  1  +  2  + … +  C20  = 2 

So, X – 1 = 2 20  – 1 


X – 1/1023 = 2 20  – 1/1023 = [(2 10 )2 – 1]/ (2 10  – 1) = 2 10  + 1. Ans.(4) 
97.  The top of the prism is an isosceles triangle, 
F 1 + a  +  a  I a 
G H  2  2  J K 
Slant surface area of all the four prisms = 4 
F  2  + 1 + 1 I  = 4 a  F 2 +  2 I = 4 a  2  + 1 
H  2  J K  G H  2  JK  e  j 
= 4 a  G 2  2  2 

Total surface area of the cube = 6a 2 

Therefore, required ratio = 
e  2  + 1 j : 6 a  = 2 e  2  + 1 j : 3  . Ans.(5) 
= 4 a 2  2 

98.  180=2 2 ×3 2 ×5 
The  highest  power  of  5  in  1!×2!×3!.............49!  is  5×[0+1+2+3+4+6+7+8+9+10]  =  5×[50]  =  250  The  highest  power  of  3  in  1!×2!×3!.............49!  is 
3×[(22×23)/2 ­ (3+7+11+12+16+20)]­22 = 530 
Highest powers of 5 and 9 are 250 and 265 respectively. 
We need not consider power of 4 as it will definetely be greater than 265 
Since b is not a multiple of 5, a is the highest power of 5, 250. Ans.(4) 
99.  a*a=a + a – a 2  = 2a – a 2  = a(2 – b) 
Similarly, b*b = b(2 – b) 
2a – a 2  < 2b – b 2 
2(a – b) < a 2  – b 2 
2(a – b) < (a – b)(a + b) 
(a – b){2 –(a + b)} < 0 
Then, 1 st  case: (a – b) < 0 and 2 – (a + b) > 0 
a < b and 2 > (a + b)

IC : PTpnrps03  15 of (17) 
1 > b > a 
2 nd  case: (a – b) > 0 and 2 – (a + b) < 0 
a > b and 2 < (a + b) 
a > b > 1 which is not true for 0.5 < (a, b) <1, hence it is rejected so only 1 st  case satisfies. Ans.(1) 
100.  We have100 £ 15k £ 10 5 
Or, 6.66 £ k £ 6666.66 or 7 £ k £ 6666 
The integer 15k is divisible by 35 if and only if k is divisible by 7. 
So, k = 7q 
we have 7£ 7q £6666 
1 £ q £952.28 
or1 £ q £ 952 
Hence, the required number of positive integers = 952. Ans.(3)

16 of 17  PTpnrps03 
Objective key 

1.(3)  2.(1)  3.(1)  4.(1)  5.(3) 


6.(5)  7.(5)  8.(4)  9.(4)  10.(2) 
11.(4)  12.(3)  13.(2)  14.(4)  15.(2) 
16.(1)  17.(4)  18.(3)  19.(3)  20.(4) 
21.(2)  22.(2)  23.(3)  24.(4)  25.(2) 
26.(2)  27.(5)  28.(1)  29.(5)  30.(1) 
31.(2)  32.(1)  33.(4)  34.(3)  35.(2) 
36.(3)  37.(2)  38.(3)  39.(4)  40.(1) 
41.(3)  42.(5)  43.(2)  44.(1)  45.(4) 
46.(3)  47.(4)  48.(4)  49.(4)  50.(3) 
51.(1)  52.(5)  53.(4)  54.(5)  55.(2) 
56.(1)  57.(2)  58.(3)  59.(3)  60.(3) 
61.(3)  62.(1)  63.(5)  64.(3)  65.(5) 
66.(2)  67.(5)  68.(1)  69.(4)  70.(2) 
71.(5)  72.(3)  73.(5)  74.(1)  75.(2) 
76.(5)  77.(4)  78.(5)  79.(4)  80.(2) 
81.(3)  82.(3)  83.(3)  84.(4)  85.(4) 
86.(3)  87.(3)  88.(3)  89.(3)  90.(4) 
91.(4)  92.(3)  93.(1)  94.(1)  95.(1) 
96.(4)  97.(5)  98.(4)  99.(1)  100.(3)

IC : PTpnrps03  17 of (17) 

You might also like